1977 USAMO Problems/Problem 5

Revision as of 14:30, 19 November 2017 by Andrewg (talk | contribs) (Formatting)

Problem

If $a,b,c,d,e$ are positive numbers bounded by $p$ and $q$, i.e, if they lie in $[p,q], 0 < p$, prove that \[(a+b +c +d +e)\left(\frac{1}{a} +\frac {1}{b} +\frac{1}{c} + \frac{1}{d} +\frac{1}{e}\right) \le 25 + 6\left(\sqrt{\frac {p}{q}} - \sqrt {\frac{q}{p}}\right)^2\] and determine when there is equality.

Solution

Fix four of the variables and allow the other to vary. Suppose, for example, we fix all but $x$. Then the expression on the LHS has the form $(r + x)(s + \frac{1}{x}) = (rs + 1) + sx + \frac{r}{x}$, where $r$ and $s$ are fixed. But this is convex. That is to say, as $x$ increases if first decreases, then increases. So its maximum must occur at $x = h$ or $x = k$. This is true for each variable.

Suppose all five are $h$ or all five are $k$, then the LHS is 25, so the inequality is true and strict unless $h = k$. If four are $h$ and one is $k$, then the LHS is $17 + 4(\frac{h}{k} + \frac{k}{h})$. Similarly if four are $k$ and one is $h$. If three are $h$ and two are $k$, then the LHS is $13 + 6(\frac{h}{k} + \frac{k}{h})$. Similarly if three are $k$ and two are $h$.

$\frac{h}{k} + \frac{k}{h} \geq 2$ with equality iff $h = k$, so if $h < k$, then three of one and two of the other gives a larger LHS than four of one and one of the other. Finally, we note that the RHS is in fact $13 + 6(\frac{h}{k} + \frac{k}{h})$, so the inequality is true with equality iff either (1) $h = k$ or (2) three of $v, w, x, y, z$ are $h$ and two are $k$ or vice versa.

See Also

1977 USAMO (ProblemsResources)
Preceded by
Problem 4
Followed by
Last Question
1 2 3 4 5
All USAMO Problems and Solutions

The problems on this page are copyrighted by the Mathematical Association of America's American Mathematics Competitions. AMC logo.png